3
$\begingroup$

Given $g\in L^2(\mathbb{R}^3)$, consider the following function ( defined for $r>0$ ): $$c(r):=\int_{\mathbb{R}^3}\frac{g(x)}{|x|^2+r}dx$$ I'm interested in the behavior of $c(r)$ for large $r$. A simple application of Cauchy-Schwarz inequality gives $$|c(r)|\lesssim r^{-\frac14}$$ However, I think that something better can be said about $c$. For example,

is it true that $c\in L^4(1,+\infty)$?

Thank you for any suggestions.

$\endgroup$
4
  • 1
    $\begingroup$ I believe $\Vert g\Vert_2 r^{-1/4}$ is correct. $\endgroup$ Sep 1, 2016 at 19:23
  • $\begingroup$ The right answer is that $\int_1^\infty c(r)^2 r^{-1/2}\,dr<+\infty$ and that is the best that can be said in the natural class of functions satisfying the assumptions that $c(r)>0$ and $rc(r)$ is increasing (those properties correspond to $g\ge 0$). So, yes, $L^4$ is guaranteed. $\endgroup$
    – fedja
    Sep 1, 2016 at 22:06
  • 1
    $\begingroup$ @T.Amdeberhan: of course you are right, i simply include in the $\lesssim$ any constant depending only on $g$. $\endgroup$
    – Capublanca
    Sep 2, 2016 at 5:10
  • $\begingroup$ @fedja: could you show me how to obtain the inequailty $\int_1^{\infty}c(r)^2r^{-1/2}dr<+\infty$? Moreover, if i understand well, you are saying that $c\in L^4$ for any $g$, cause we can separate positive and negative part of $g$ and applying your result. It's right? $\endgroup$
    – Capublanca
    Sep 2, 2016 at 5:21

3 Answers 3

5
$\begingroup$

Once you asked, here goes. Let $h(r)=c(r^2)$. Then you need to look at the operator $$ \Psi: f\mapsto \int_0^\infty f(\rho)\frac{\rho}{\rho^2+r^2}\,d\rho. $$ Note two things:

1) The kernel is positive and dominated by $1/(r+\rho)$ (just consider the cases $\rho\le r$ and $\rho\ge r$ separately. Hence, it is not worse than the classical Hilbert operator and it is bounded in $L^2$ (by the Schur test with $\varphi(\rho)=\rho^{-1/2}$, say).

2) If $h$ is any non-negative $L^2$ function such that $h(r)r^2$ is non-decreasing, then $$ (\Psi h)(r)\ge\int_r^{2r}\frac{sh(s)}{s^2+r^2}\ge\int_r^{2r}\frac{r^2h(r)/s}{s^2+r^2}\ge \frac 1{10}h(r) $$ so nothing better can be said.

The remaining change of variable should be clear.

$\endgroup$
1
  • $\begingroup$ Thank you! This is simpler than my approach and you get a bit more than $c\in L^4$. $\endgroup$
    – Capublanca
    Sep 2, 2016 at 14:16
5
$\begingroup$

Let $h(s)$ be the average of $g(x)$ over the sphere $|x|=s$. Thus

$$c(r) = \int_0^\infty \dfrac{4 \pi s^2 h(s) \; ds}{s^2 + r}$$

where $$ \int_0^\infty 4 \pi s^2 h(s)^2 \; ds \le \|g\|_2^2 < \infty$$ So Cauchy-Schwarz says $$ c(r) \le 4 \pi \|s/(s^2+r)\|_2 \|s h(s)\|_2 \le \dfrac{\pi}{r^{1/4}} \|g\|_2 $$ with equality for $g(x) = 1/(|x|^2 + r)$.

This won't necessarily give the correct asymptotics as $r \to \infty$ for any particular $h$, but we can't improve the power of $r$ in general. Consider sequences $r_n \to \infty$ and $(a_n)_n \in \ell^1$, with $a_n > 0$, and
$$ h(s) = \sum_n a_n \dfrac{r_n^{1/4}}{s^2 + r_n}$$ We have $s h(s) \in L^2$ and $$ c(r) = \sum_n a_n {r_n}^{1/4} \int_0^\infty \dfrac{4\pi s^2 \; ds}{(s^2+r)(s^2 + r_n)} = \sum_n a_n \dfrac{2\pi^2 {r_n}^{1/4}}{\sqrt{r} + \sqrt{r_n}} $$ In particular $c(r_n) \ge \pi^2 a_n r_n^{-1/4} $. Given $\epsilon > 0$ and $(a_n)$, we can let $r_n = a_n^{-1/\epsilon}$, and then $c(r_n) \ge \pi^2 r_n^{\epsilon - 1/4}$, so that $$ \limsup_{r \to \infty} c(r) r^{1/4-\epsilon} > 0$$

$\endgroup$
5
  • $\begingroup$ In other words, the only thing known about $g$ is that $\|g\|_2 < \infty$, then $c \not\in L^4(1,\infty)$, because at least for $g(x) = 1/(|x|^2+r)$ it decays too slowly. $\endgroup$ Sep 1, 2016 at 21:31
  • $\begingroup$ That's not clear. For a particular $r_n$ and $g_n(x) = 1/(|x|^2 + r_n)$, $c(r) = 2 \pi^2 r_n^{1/4}/(\sqrt{r} + \sqrt{r_n})$ is in $L^4$. $\endgroup$ Sep 1, 2016 at 21:47
  • $\begingroup$ Ah, I fell into a trap! I forgot that $g(x)$ depends only on $x$ and is not allowed to depend on $r$. $\endgroup$ Sep 1, 2016 at 23:01
  • $\begingroup$ @RobertIsrael: thank you for your anwer. Now it's clear that the power of $r$ can't be improved in general. But this still don't exclude the case that $c\in L^4$, right? $\endgroup$
    – Capublanca
    Sep 2, 2016 at 5:26
  • $\begingroup$ @IgorKhavkine: Indeed, i think that the independence of $g$ on $r$ is crucial here. $\endgroup$
    – Capublanca
    Sep 2, 2016 at 5:28
0
$\begingroup$

I think i have a simple proof that $c\in L^4$, i give a sketch of it.

We can suppose $g$ positive (separating positive and negative part), then we can pass to its radial rearrangement (this preserves the $L^2$ norms).

Passing to polar coordinates, we get $$c(r)=\int_0^{\infty}g(\rho)\cdot\frac{\rho^2}{\rho^2+r}d\rho$$ Since $\rho g(\rho)\in L^2(0,\infty)$, it`s natural to consider the operator $$\Psi:f\mapsto\int_0^{\infty}f(\rho)\cdot\frac{\rho}{\rho^2+r}d\rho$$

and to show that it`s bounded from $L^2(0,\infty)$ to $L^4(0,\infty)$.

We can do this using Marcinkiewicz interpolation theorem. For example, if $f\in L^{3/2}$, Holder inequality gives

$$|\Psi(f)(r)|\leq\Vert f\Vert_{3/2}\left\Vert\frac{\rho}{\rho^2+r}\right\Vert_3\lesssim (1+r)^{-3}\Vert f\Vert_{3/2}$$ This means that $$\Psi:L^{3/2}\rightarrow L^{3,\infty}$$ In the same way, we can show that $$\Psi:L^{4}\rightarrow L^{8,\infty}$$ Then by Marcinkiewicz theorem $$\Psi:L^{2}\rightarrow L^{4}$$ and we are done.

Do you think the proof it's correct? In that case i will expand it. Thank you.

$\endgroup$

Your Answer

By clicking “Post Your Answer”, you agree to our terms of service and acknowledge you have read our privacy policy.

Not the answer you're looking for? Browse other questions tagged or ask your own question.